• 沒有找到結果。

0, there exists N ∈ N such that if n &gt

N/A
N/A
Protected

Academic year: 2022

Share "0, there exists N ∈ N such that if n &gt"

Copied!
20
0
0

加載中.... (立即查看全文)

全文

(1)

Sequences

Definition A sequence {pn} in X is a function f : N → X such that f(n) = pn ∈ X.

Definition The sequence {pn} converges in X if there exists p ∈ X such that for each  > 0, there exists N ∈ N such that if n > N then d(p, pn) < . We say {pn} converges to p. Or equivalently, p is the limit of {pn}.

Notation pn→ p ⇐⇒ lim

n→∞{pn} = p.

Definitions

(a) If {pn} does not converge to any point in X then it diverges.

(b) The range of {pn} is {x ∈ X | x = pn for some n}.

(c) We say {pn} isbounded in X if the range of {pn} is bounded in X.

True/False Questions Let {pn} be a sequence in a metric space X.

(a) If pn → p and pn → p0, then p = p0. True.

(b) If {pn} is bounded, then {pn} converges. False.

(c) If pn converges, then {pn} is bounded. True.

(d) If pn → p, then p is a limit point of the range of {pn}. False.

(e) If p is a limit point of E ⊆ X, then there exists some sequence {pn} such that pn → p. True.

(f) pn→ p if and only if every neighborhood of p contains all but finitely many terms in {pn}.

True.

Remark “All but finitely many” is equivalent to “Almost all.”

Solution

a) Let  > 0. Then there exists N1 ∈ N such that n > N1, d(p, pn) < 

2. Similarly there exists N2 ∈ N such that n > N2, d(p0, pn) < 

2. Then for n > max (N1, N2):

d(p, p0) ≤ d(p, pn) + d(p0, pn) < 2

2 = .

Thus d(p, p0) <  for all  > 0. Thus d(p, p0) = 0. Hence p = p0. b) Consider Ex. (4) where there is oscillation between two points.

c) Suppose pn → p. Since 1 > 0 (our choice of ), there exists N ∈ N such that for n > N implies d(pn, p) < 1. Let r = max (1, d(p, p1), . . . , d(p, pN)) + 1. Then pn∈ B(p, r) for all n ∈ N.

d) Consider Ex. (3) where the sequence is simply one point.

e) p ∈ E0. For all n ∈ N such that pn ∈ E such that d(pn, p) < n1 and pn 6= p. There for a sequence {pn}. Let  > 0. Choose N > 1

. Then for each n > N , we have d(p, pn) < 1 n < 1

N < .

(2)

Thus this sequences {pn} converges to p as desired.

f) (⇒) Suppose pn → p. Given B(p, ), there exists N ∈ N such that when n > N, it fol- lows that d(p, pn) < , i.e. pn∈ B(p, ), leaving only finitely many points, p1 through pnpossible.

(⇐) For all  > 0, B(p, ) contains almost all {pn}. For  > 0, let m = max{n ∈ N | pn6∈ B(p, )}.

Then n > m implies pn∈ B(p, ), i.e. d(pn, p) < .

Theorem (Limit Laws) Suppose {sn}, {tn} are sequences in C, and lim

n→∞sn = s, lim

n→∞tn = t.

Then (a) lim

n→∞(sn+ tn) = s + t.

(b) lim

n→∞(csn) = cs for all c ∈ C.

(c) lim

n→∞(c + sn) = c + s for all c ∈ C.

(d) lim

n→∞(sntn) = st.

(e) lim

n→∞

1 sn

= 1

s provided sn6= 0 for all n ∈ N, and s 6= 0.

Outline of the Proof (a) Let  > 0. Since lim

n→∞sn = s and lim

n→∞tn= t, there exists N1, N2 ∈ N such that if n ≥ N1, and if n ≥ N2 then |sn− s| < 

2 and |tn− s| < 

2, respectively. This imples that if n ≥ max (N1, N2), then

|(sn+ tn) − (s + t)| = |(sn+ tn) − (s + t)|

= |sn− s| + |tn− t|

< .

(b) Let  > 0. Since lim

n→∞sn= s, there exists N ∈ N such that if n ≥ N, then |sn− s| <  1 + |c|. This imples that if n ≥ N, then |csn− cs| ≤ |c| |sn− s| < |c| 

1 + |c| < .

(d) Let  > 0. Choose k = max(|s|, |t|, 1, ). Since lim

n→∞sn = s and lim

n→∞tn = t, there exists N1, N2 ∈ N such that if n ≥ N1, and if n ≥ N2 then |sn− s| < 

3k and |tn− s| < 

3k, respectively.

This imples that if n ≥ max (N1, N2), then

|sntn− st| = |(sn− s)(tn− t) + s(tn− t) + t(sn− s)|

≤ 2

9k2 + |s|

3k +|t|

3k

≤  9k + 

3+ 

3 since k ≥ |s|, |t|,  =⇒ 1 ≥ |s|

k , |t|

k ,  k

≤  9+ 

3+ 

3 since k ≥ 1

< .

(3)

(e) Let  > 0. Since lim

n→∞sn = s 6= 0, there exists N1, N2 ∈ N such that if n ≥ N1, then |sn− s| < 1

2|s| =⇒ |s| − |sn| ≤ |sn− s| < 1

2|s| =⇒ 1

2|s| ≤ |sn| if n ≥ N2, then |sn− s| < 1

2|s|2, respectively.

This imples that if n ≥ max (N1, N2), then

1 sn − 1

s

=

sn− s sns

< 2

|s|2 |sn− s|

< .

Subsequences

Definition Let {pn} be a sequence in X and let {ni} be a sequence of natural numbers such that n1 < n2 < n3 < . . . . Then {pni} is a subsequence of {pn}.

Examples

(a) Let {pn} = {1, π,12, π, 13, π, . . . }. Notice this does not converge. But a subsequence does converge. For example, only the π terms. As do the other terms.

(b) Notice that {12,23,34,45, . . . } converges for 1. Notice that any subsequence converges. By property (f ) on Page 1, almost all terms of the subsequence are contained in any neighbor- hood of the limit.

Remarks

(a) Note that if pn → p and {pnk} is any subsequence of {pn}, then pnk → p.

(b) Question: Must every sequence contain a convergent subsequence?

No. Consider the sequence {1, 2, 3, . . . }, and note that there is no convergent subsequence of this sequence.

Recall that an ordered field F is said to have the least-upper-bound property if every nonempty, bounded above subset E of F , the sup E ∈ E0 exists in F. For general metric spaces without the least-upper-bound property, the following generalizations hold.

Theorem

(a) In a compact metric space X, every sequence contains a subsequence converging to a point in X, i.e. if {pn} is a sequence in a compact metric space, then there exists a subsequence {pnj} of {pn} converges to a point p ∈ X.

Proof The proof is simmilar to the proof of Theorem 2.41 (b) =⇒ (c) on p40.

(b) (Bolzano-Weierstrass) Every bounded sequence in Rkcontains a convergent subsequence.

Proof Let R = {pn | n ∈ N} = the range of {pn}.

(4)

Suppose R is a finite subset of Rk, there exists a point p ∈ R and a subsequence {nj | j ∈ N} of {n | n ∈ N} such that

pnj = p ∀ j ∈ N by the pigeon-hole principle. Since

j→∞lim pnj = p, pnj is a convergent subsequence of {pn}.

Suppose R is a infinite subset of Rk, since R is bounded, there exists a k-cell I such that R = {pn | n ∈ N} is an infinite subset of the compact set I.

Bisecting sides of I into 2k k-cells {Qi}2i=1k with equal sizes such that

I =

2k

[

i=1

Qi.

Let I1 ∈ {Qi}2i=1k such that

I1∩ R contains infinitely many elements and diam (I1) = 1

2diam (I).

Next subdivide I1 and continue the process to obtain a sequence of k-cells such that I ⊃ I1 ⊃ I2 ⊃ · · ·

Ij∩ R contains infinitely many elements for each j ∈ N diam (Ij) = 1

2diam (Ij−1) = · · · = 1

2j diam (I) → 0 as j → ∞

For each j ∈ N, since Ij contains infinitely many elements of {pn}, there is a subsequence {nj} such that pnj ∈ Ij∩ R. Next, since lim

j→∞diam (Ij) = 0,

\

j=1

Ij contains exactly one point, say p ∈ Rk. This implies that

{p} =

\

j=1

Ij and 0 ≤ lim

j→∞d(pnj, p) ≤ lim

j→∞diam (Ij) = 0 =⇒ lim

j→∞pnj = p.

Theorem Let X be a metric space and let {pn} be a sequence in X. Suppose that E = {pn | n ∈ N} is the set of points determined by {pn} and E is the set of all subsequential limits of {pn} given by

E = {p ∈ X | ∃ a subsequence {pni} ⊂ {pn} such that lim

i→∞pni = p}.

Then E is closed, i.e. E0

⊆ E.

(5)

Proof

Suppose E = {q1, . . . , qm} contains finitely many points of X.

=⇒ E contains finite number of points

=⇒ E0

= ∅ ⊆ E

=⇒ E is closed.

Suppose E contains infinitely many points of X and assume that E0

6= ∅.

=⇒ Given q ∈ E0

, let pn1 ∈ {pn} such that pn1 6= q

=⇒ δ = d(q, pn1) > 0.

Since q ∈ E0

=⇒ ∃ x1 ∈ Nδ/2(q) ∩ E\ {q} and since x1 ∈ E

=⇒ ∃ n2 > n1 such that pn2 ∈ Nδ/2(x1) ∩ E.

=⇒ d(q, pn2) ≤ d(q, x1) + d(x1, pn2) < δ.

Similarly, ∃ x2 ∈ Nδ/22(q) ∩ E\ {q} and since x2 ∈ E

=⇒ ∃ n3 > n2 > n1 such that pn3 ∈ Nδ/22(x2) ∩ E.

=⇒ d(q, pn3) ≤ d(q, x2) + d(x2, pn3) < δ/2.

q pn1

x1 pn2 δ

δ/2

δ/2

x2 pn3

δ/4 δ/4

For each k ≥ 2, suppose pn1, . . . , pnk are chosen such that d(pnj, q) < δ/2j−2 for each 2 ≤ j ≤ k, then we choose

xk ∈ Nδ/2k(q) ∩ E\ {q} and pnk+1 ∈ Nδ/2k(xk) ∩ E

(6)

such that

d(q, pnk+1) ≤ d(q, xk) + d(xk, pnk+1) < δ/2k−1.

pnk

xqk

pnk+1 δ/2k−2

δ/2k−1

δ/2k

δ/2k

Since lim

k→∞d(q, pnk) ≤ lim

k→∞δ/2k−2 = 0, {pnk} is a subsequence of {pn} converging to q and hence q ∈ E. Since q is an arbitrary point from q ∈ E0

, we have shown E0

⊆ E and E is closed.

Cauchy Sequences

Question How to tell if {pn} converges if you don’t know the limit already?

Definition The sequence {pn} is a Cauchy sequence if for each  > 0, there exists N ∈ N such that if m, n > N then it is implied that d(pm, pn) < .

Definition Let E be a subset of a metric space X = (X, d). The diameter of E is defined by diam E = sup{d(p, q) | p, q ∈ E}.

Remarks

(a) Note that 0 ≤ diam E ≤ ∞.

(b) Let {pn} be a sequence in X and let EN = {pn | n ≥ N }. Then {pn} is a Cauchy sequence if and only if

lim

N →∞diam EN = 0.

(c) Let E be a subset of a metric space X = (X, d). Then diam ¯E = diam E.

Proof Since E ⊆ ¯E, we have diam E ≤ diam ¯E.

Given  > 0 and for any p, q ∈ ¯E, since ¯E = E ∪ E0, there exist p0, q0 ∈ E such that d(p, p0) < 

2 and d(q, q0) <  2.

(7)

This implies that

d(p, q) ≤ d(p, p0) + d(p0, q0) + d(q, q0) ≤ 

2 + diam E + 

2 = diam E +  ∀ p, q ∈ ¯E, and

diam ¯E ≤ diam E +  =⇒ diam ¯E ≤ diam E by letting  → 0+. Hence we have diam ¯E = diam E.

(d) If Kn is a sequence of compact sets in X such that Kn ⊃ Kn+1 for all n ∈ N and if

n→∞lim diam Kn= 0,

then

\

n=1

Kn consists of exactly one point.

Theorem

(a) In any metric space X, every convergent sequence is a Cauchy sequence, i.e. if {pn} ⊂ X converges, then it is Cauchy.

Proof Suppose that lim

n→∞pn= p. Then ∀  > 0 there exists N such that n > N d(pn, p) <  2. So for m, n > N we know

d(pn, pm) ≤ d(pn, p) + d(pm, p)

<  2 + 

2

= .

Remark Not every Cauchy sequence converges. To see this, let X = Q. Let pn be the smallest such that m

n > π. Then {pn} is Cauchy, but does not converge in Q (in R converge to π).

(b) If X is a compact metric space and if {pn} is a Cauchy sequence in X, then {pn} converges to some point of X.

Proof Consider the set S defined by

S = {pn| n ∈ N}, i.e. the range set of the map p : N → X defined by p(n) = pn. Case 1 S contains finitely many points, say S = {pn1, . . . , pnm}.

Let

δ = min{d(pni, pnj) | 1 ≤ i < j ≤ m}.

Given  > 0 such that  < δ, since {pk} is Cauchy, there exists N ∈ N such that if k, l ≥ N then d(pk, pl) <  =⇒ if k, l ≥ N then d(pk, pl) < δ.

Suppose ∃ k, l ≥ N such that pk 6= pl, then we must have

δ ≤ d(pk, pl) < δ =⇒ δ < δ which is a contradiction.

Hence we have

∀ k, l ≥ N, pk = pl =⇒ if k ≥ N, then pk = pnj for some 1 ≤ j ≤ m.

(8)

Therefore we have

lim

k→∞pk = pnj for some 1 ≤ j ≤ m, i.e. {pk} converges to pnj ∈ X.

Case 2 S contains infinitely many points.

Since X is compact and S is an infinite subset of X,

X is closed, S0 6= ∅ and S0 ⊂ X0 = X =⇒ ∃ p ∈ S0 ⊂ X.

This implies that for each k ∈ N, there exists pnk ∈ B1/k(p) ∩ S \ {p}. Thus we obtain a subsequence {pnk} of {pn} converging to p. This imples that

∀  > 0, ∃ N1 ∈ N such that if k ≥ N1, then d(pnk, p) < .

Also since {pn} is a Cauchy sequence,

with the given  > 0, ∃ N2 ∈ N such that if n, m ≥ N2, then d(pn, pm) < .

Let N = max{N1, N2}. Since

d(pn, p) ≤ d(pn, pnn) + d(pnn, p) < 2 ∀ n ≥ N =⇒ lim

n→∞pn = p, i.e. {pn} converges to p ∈ X.

(c) In Rk, every Cauchy sequence converges.

Definition A metric space X is complete if every Cauchy sequence converges to a point in X.

Remarks

(a) Compact metric spaces are complete.

Proof Let {xi} be Cauchy in X. Then {xi} has a convergent subsequence. So there exist {xnk} converging to x ∈ X. Fix  > 0. Cauchy implies there exists N ∈ N such that i, j > N then d(xi, xj) < 2. By convergence of {xnk}, there exists N0 ∈ N such that whenever nk> N0, then d(x, xnk) < 2. Let N00 = max (N, N0). Then i, nk> N00 implies

d(x, xi) ≤ d(x, xnk) + d(xnk, xi)

<  2+ 

2

= .

Hence the sequence itself converges to x and therefore X is complete.

(b) Rk is complete but Q is not.

(c) Let X be the ordered field R. Then X has the leat-upper-bound property if and only if every Cauchy sequence converges to a point in X. Thusan ordered field X is complete if it has the least-upper-bound property.

Proof

(=⇒) Let S be a nonempty, bounded from above subset of X = R. Choose a sufficiently large M > 0 such that

M is an upper bound of S and [−M, M ] ∩ S 6= ∅.

(9)

Let I1 = [−M, M ]. Divide I1 into 2 equal length subintervals and let I2 be the closed subinterval such that

the right endpoint of I2 is an upper bound of S and I2∩ S 6= ∅.

Continuing this process, we obtain a sequence of closed intervals {In} such that the right endpoint of In is an upper bound of S, In∩ S 6= ∅ ∀ n ≥ 1.

and

I1 ⊃ I2 ⊃ · · · ⊃ In⊃ · · · ; lim

n→∞|In| = lim

n→∞

M 2n−2 = 0, For each n ∈ N, since In∩ S 6= ∅, let xn be a point in In∩ S.

Since lim

n→∞|In| = lim

n→∞

M

2n−2 = 0, {xn} is a Cauchy sequence and hence it converges to a point, say x, in X = R.

(⇐=) Let {xn} be a Cauchy sequence in X = R. Since {xn} is bounded, there exists x ∈ X = R and a closed interval I = [−M, M] such that

x = sup{xn | n ∈ N} and {xn} ⊂ I.

Since I is compact and {xn} is a Cauchy sequence, we have

n→∞lim xn = x.

Theorem Every metric space (X, d) has a completion (X?, ∆). In other words, (X?, ∆) is a complete metric space containing in X.

Let X? = {Cauchy sequences in X}/ ∼. We will say:

{pn} ∼ {p0n} ⇐⇒ lim

n→∞d(pn, p0n) = 0.

Let P, Q ∈ X?. Then P = [{pn}], and Q = [{p0n}]. We define:

∆(P, Q) = lim

n→∞d(pn, qn).

We claim (X?, ∆) is a complete metric space and (X, d) is isometrically embedded in (X?, ∆).

In other words, there is an injection i : X ,→ X? such that d(p, q) = ∆(i(p), i(q)).

Remarks

(a) This is the other construction of R. But Dedekind cuts are more hardcore.

(b) Example If X = Q, then X? = R. In particular, X? is isometrically isomorphic to R. In other there is a distance preserving bijection.

Definition A sequence {sn} in R is monotonically increasing if sn ≤ sn+1 for all n. Similarly {sn} ismonotonically decreasing if sn ≥ sn+1.

(10)

Theorem Suppose {sn} is a monotonic sequence of real numbers. Then {sn} converges if and only if it is bounded.

Proof

(⇐=) Suppose sn ≤ sn+1 (the proof is analogous in the other case).

Let

E = {sn| n ∈ N} = the range of {sn}.

If {sn} is bounded, let

s = sup E = sup{sn| n ∈ N}.

Then

sn ≤ s ∀ n ∈ N.

For every  > 0, since s −  < s and there is an integer N such that s −  < sN ≤ s,

for otherwise s −  would be an least-upper-bound of E. Since sn+1≥ sn for all n ∈ N, therefore implies

if n ≥ N, then s −  < sN ≤ sn ≤, which shows that {sn} converges to s.

(=⇒) Suppose that s = lim

n→∞sn. By taking  = 1, there exists K ∈ N such that if n ≥ K, then |sn| − |s| ≤ |sn− n| <  = 1 =⇒ |sn| < 1 + |s| ∀n ≥ K.

Thus

if L = max{|s1|, . . . , |sK−1|, 1 + |s|} =⇒ |sn| ≤ L ∀ n ∈ N.

This proves that {sn} is bounded.

Remarks

(a) Monotone Sequence Property Let F be an ordered field. We say F has the monotone sequence property if every bounded above monotonically increasing sequence converges.

(b) Let X be the ordered field R. Then X has the least-upper-bound property if and only if X has the monotone sequence property. Thus an ordered field X is complete if it has the monotone sequence property.

Proof

(⇐=) Let S be a nonempty, bounded from above subset of X = R. Choose a sufficiently large M > 0 such that

M is an upper bound of S and [−M, M ] ∩ S 6= ∅.

Let I1 = [−M, M ]. Divide I1 into 2 equal length subintervals and let I2 be the closed subinterval such that

the right endpoint of I2 is an upper bound of S and I2∩ S 6= ∅.

Continuing this process, we obtain a sequence of closed intervals {In} such that the right endpoint of In is an upper bound of S, In∩ S 6= ∅ ∀ n ≥ 1.

(11)

and

I1 ⊃ I2 ⊃ · · · ⊃ In⊃ · · · ; lim

n→∞|In| = lim

n→∞

M 2n−2 = 0,

For each n ∈ N, by letting xn be the left endpoint of In, we obtain a bounded above monotonically increasing sequence {xn}. Hence there exists x ∈ X = R such that

n→∞lim xn = x.

Since lim

n→∞|In| = lim

n→∞

M

2n−2 = 0 and, for each n ≥ 1, the right endpoint of In is an upper bound of S, x = sup{xn| n ∈ N} ∈ X = R.

(=⇒) Let {xn} be a bounded above monotonically increasing sequence in X = R. Since {xn} is bounded above monotonically increasing, there exists x ∈ X = R such that

n→∞lim xn = x.

Upper and Lower Limits

Definition Let {sn} be a sequence of real numbers.

(a) lim

n→∞sn = ∞ iff ∀ M ∈ R, ∃ N ∈ N such that if n ≥ N then sn ≥ M.

(b) lim

n→∞sn = −∞ iff ∀ M ∈ R, ∃ N ∈ N such that if n ≥ N then sn ≤ M.

Definition 3.16 Let {sn} be a sequence of real numbers and let E be the set of subsequential limits of {sn}defined by

E =n

x ∈ (−∞, ∞) ∪ {±∞} | ∃ {snk} ⊂ {sn} such that lim

k→∞snk = xo . Let

lim sup

n→∞

sn = s = sup E, lim inf

n→∞ sn = s = inf E.

The numbers s, s are called the upper and lower limits of {sn}; we use the notation lim sup

n→∞

sn = s, lim inf

n→∞ sn= s.

Theorem 3.17 Let {sn} be a sequence of real numbers and let E be the set of subsequential limits of {sn}. Then s = sup E and s = inf E have the following properties:

(a) s, s ∈ E, i.e. there exist subsequences {snk}, {smj} of {sn} such that lim

k→∞snk = s, lim

j→∞smj = s. Proof

If s = ∞, then E is not bounded above; hence {sn} is not bounded above, and there is a subsequence {snk} such that lim

k→∞snk = ∞.

(12)

If s ∈ R = (−∞, ∞), then E is bounded above; and at least one subsequential limit exists, so that (a) follows from the Weierstrass Theorem.

If s = −∞, then E contains only one element, namely −∞, and there is no subsequential limit. Hence for any M ∈ R, there exist at most a finite number of values of n such that sn > M which implies that lim

n→∞= −∞.

(b) If x > s ≥ s > y, there exists an N ∈ N such that n ≥ N implies x > sn > y, i.e. for each

 > 0, there exists an N ∈ N such that

if n ≥ N then s−  < sn< s+ .

s+  s − 

Proof For each  > 0, if there are infinitely many sn’s lying in (−∞, s− ] ∪ [s+ , ∞), then there exists a subsequence {snk} lying completely in (−∞, s− ] or [s+ , ∞).

This implies that there exists a subsequential limit x ∈ (−∞, s− ] ∪ [s+ , ∞) ∪ {±∞}

which is a contradiction to the definition of s = inf E or that of s = sup E.

Hence there are only finite number of xn’s lying in (−∞, s− ] ∪ [s+ , ∞).

Remarks

(a) If k ≥ l ∈ N, then {sm | m ≥ k} ⊆ {sm | m ≥ l} and

inf{sm | m ≥ l} ≤ inf{sm | m ≥ k} ≤ sup{sm | m ≥ k} ≤ sup{sm | m ≥ l},

i.e. sup{sm | m ≥ k} is monotone decreasing in k, and inf{sm | m ≥ k} is monotone increasing in k.

(b) If s, s ∈ E, and {snk}, {smj} are subsequences of {sn} such that

k→∞lim snk = s, lim

j→∞smj = s =⇒ lim inf

k→∞ snk = s = lim sup

k→∞

snk, lim inf

j→∞ smj = s = lim sup

j→∞

smj. For each  > 0, since there only exist finite number of n ∈ N such that sn > s +  or sn < s− , hence we have

s−  ≤ inf{sm | m ≥ n} ≤ sup{sm | m ≥ n} ≤ s+  for sufficiently large n

=⇒ s−  ≤ lim

n→∞inf{sm | m ≥ n} ≤ lim

n→∞sup{sm | m ≥ n} ≤ s+  ∀  > 0

=⇒ lim

→0+(s− ) ≤ lim

n→∞inf{sm | m ≥ n} ≤ lim

n→∞sup{sm | m ≥ n} ≤ lim

→0+(s+ )

=⇒ s ≤ lim

n→∞inf{sm | m ≥ n} ≤ lim

n→∞sup{sm | m ≥ n} ≤ s.

On the other hand for each l ∈ N, since {smj | j ≥ l} and {snk | k ≥ l} are subsequences of {sn | n ≥ ml} and {sm | m ≥ nl} respectively, we have

inf{sn| n ≥ ml} ≤ smj and snk ≤ sup{sm | m ≥ nl} ∀ j, k ≥ l

=⇒ inf{sn| n ≥ ml} ≤ lim

j→∞smj = s and s = lim

k→∞snk ≤ sup{sm | m ≥ nl} ∀ l ∈ N

=⇒ inf{sn| n ≥ ml} ≤ s ≤ s ≤ sup{sm | m ≥ nl} ∀ l ∈ N

=⇒ lim

l→∞inf{sn | n ≥ ml} ≤ s ≤ s ≤ lim

l→∞sup{sm | m ≥ nl}

=⇒ lim

n→∞inf{sm | m ≥ n} ≤ s ≤ s ≤ lim

n→∞sup{sm | m ≥ n}.

(13)

Hence

lim sup

n→∞

sn= s = lim

n→∞sup{sm | m ≥ n} = inf{sup{sm | m ≥ n} | n ∈ N}, lim inf

n→∞ sn= s = lim

n→∞inf{sm | m ≥ n} = sup{inf{sm| m ≥ n} | n ∈ N}.

(c) If x > s = lim

n→∞sup{sm | m ≥ n} and lim

n→∞inf{sm | m ≥ n} = s > y, there exists an N ∈ N depending on x − s > 0 and s − y > 0 such that if n ≥ N then

max{| sup{sm | m ≥ n} − s|, |s− inf{sm | m ≥ n}|} < min{x − s, s− y} ∀ n ≥ N

=⇒ sup{sm | m ≥ n} − s < x − s and s− inf{sm | m ≥ n} < s− y ∀ n ≥ N

=⇒ sn≤ sup{sm | m ≥ n} < x and sn≥ inf{sm | m ≥ n} > y ∀ n ≥ N

=⇒ y < inf{sm | m ≥ n} ≤ sn≤ sup{sm | m ≥ n} < x ∀ n ≥ N.

Examples

(a) Let {sn} = Q, be a sequence containing all rationals. Then every real number is a subse- quential limit, and

lim sup

n→∞

sn= ∞, lim inf

n→∞ sn = −∞, i.e. the set of subsequential limits of {sn} is E = R ∪ {±∞}.

(b) Let sn= (−1)n

1 + (1/n). Then

lim sup

n→∞

sn = 1, lim inf

n→∞ sn = −1.

(c) For a real-valued sequence {sn},

n→∞lim sn = s ⇐⇒ lim sup

n→∞

sn = lim inf

n→∞ sn = s.

Theorem If sn ≤ tn for n ≥ N, where N is fixed, then lim inf

n→∞ sn≤ lim inf

n→∞ tn, lim sup

n→∞

sn ≤ lim sup

n→∞

tn.

Some Special Sequences

Theorem

(a) If p > 0, then lim

n→∞

1 np = 0.

Proof For each  > 0, there exists n ∈ N such that n > 1

1/p ⇐⇒ np > 1

 ⇐⇒  > 1 np = | 1

np − 0|.

(14)

(b) p > 0, then lim

n→∞

n

p = 1.

Proof If p > 1, by letting xn = √n

p − 1 > 0, then 1 + n xn≤ (1 + xn)n= p =⇒ xn ≤ p − 1

n =⇒ lim

n→∞xn = 0 ⇐⇒ lim

n→∞

n

p = 1.

If p < 1, by letting p = 1

q for some q > 1, we have

n→∞lim

n

q = 1 =⇒ lim

n→∞

1

n

q = 1 =⇒ lim

n→∞

n

p = lim

n→∞

1

n

q = 1.

(c) lim

n→∞

n

n = 1.

Proof Let xn = √n

n − 1. Then xn≥ 0 for each n ∈ N and n = (1 + xn)n≥ n(n − 1)

2 x2n =⇒ 0 ≤ xn

r 2

n − 1 for n ≥ 2 =⇒ lim

n→∞xn = 0.

(d) If p > 0 and α ∈ R, then lim

n→∞

nα

(1 + p)n = 0.

Proof Let k ∈ N such that k > α. If n ∈ N and n

2 > k, then (1 + p)n>n

k



pk = n(n − 1) · · · (n − k + 1)

k! pk> nkpk

2kk! =⇒ 0 < nα

(1 + p)n < 2kk!

pk nα−k. (e) If |x| < 1, then lim

n→∞xn= 0.

Proof Take α = 0 and p > 0 such that |x| = 1

1 + p as in (d).

Series

Question What does this mean?

1 + 1 4+ 1

9+ 1 16+ 1

25+ · · · = π2 6 .

Remark Sum of natural numbers to negative even powers always has a nice form.

Consider also:

1 − 1 + 1 − 1 + 1 − 1 + . . .

You can associate these differently and get different limits. This tells us we cannot assume associativity in infinite sums.

Notation Let {an} be a real sequence. Then:

j

X

n=i

an = ai+ ai+1+ · · · + aj, when i < j ∈ N.

(15)

Definition The nth partial sum of {ak} is sn=

n

X

k=1

ak.

Remark {sn} is a real sequence. Sometimes {sn} is called an infinite series.

Definition If sn → s we write

X

n=1

an = lim

n→∞

n

X

k=1

ak = s

Question When does an infinite series converge? When its sequence of partial sums converge.

Example Does

X

n=1

1

n converge?

Consider partial sums where sn=

n

X

k=1

1

k. We use the Cauchy criterion. For m < n, then d(sm, sn) = sn− sm

= sm+1+ sm+2+ · · · + sn

= 1

m + 1 + 1

m + 2 + · · · + 1

n ≥ n − m n .

The inequality comes from observing that all the terms in the sum are less than or equal to 1 n. Thus s2n − sn ≥ 2n − n

2n = 1

2. Therefore this sequence is not Cauchy. Hence {sn} does not converge.

Cauchy Criterion for series X

an converges if and only if for all  > 0 there exists N ∈ N such that m ≥ n > N implies

m

X

k=n

ak

< .

Corollary (Divergence Test) If X

an converges, then lim

n→∞an = 0.

Remarks (a) X 1

n is called the harmonic series.

(b) The corollary’s converse is not true (counter-example is harmonic series).

Theorem If an ≥ 0, then X

an converges if and only if partial sums {sn} form a bounded sequence.

Proof Since ak ≥ 0 for all k ∈ N, {sn} is monotonically increasing. Thus {sn} converges, i.e.

Xan converges, if and only if {sn} is bounded.

(16)

Comparison Test 1. If X

cn converges and |an| ≤ cn for almost all n, then X

an converges.

2. If X

dn diverges to +∞ and an ≥ dn for almost all n, then X

an diverges as well.

Proof

1. Let  > 0. Since X

cn converges, it satisfies Cauchy Criterion. Thus there exists N ∈ N such that m ≥ n ≥ N implies:

m

X

k=n

ck

m

X

k=n

ck

< .

Thus

m

X

k=n

ak

m

X

k=n

|ak| ≤

m

X

k=n

ck.

This follows from the assumption that |an| ≤ cn for almost all n so let N be at least larger than the last n for which cn < |an|. The resulting inequality satisfies the Cauchy Criterion and thusX

an converges.

2. Follows from (a) via contrapositive. (Also, partial sums form a bounded sequence.)

Geometric Series If |x| < 1, then

X

n=0

xn= 1 1 − x. If |x| ≥ 1, then X

xn diverges.

Proof If x 6= 1, let sn =

n

X

k=0

xk = 1 + x + · · · + xn. Then sn = 1 − xn+1

1 − x by multiplying both sides by 1 − x. Thus it follows:

n→∞lim sn = 1

1 − x if |x| < 1

If |x| > 1, then {sn} does not converge. Similarly if x = ±1, use the divergence test to verify {sn} does not converge.

Example

X

n=0

1

n! converges. Notice

sn= 1 + 1 + 1 2! + 1

3! + 1

4! + · · · + 1 n!

≤ 1 + 1 + 1 2+ 1

22 + 1

23 + · · · + 1 2n−1

< 3.

Thus it is bounded and since each term is nonnegative, it is monotonically increasing. Thus it converges.

(17)

Definition e =

X

n=0

1 n!. Theorem e = lim

n→∞(1 + 1 n)n. Proof

Let sn=

n

X

k=0

1

k!, tn = 1 + 1 n

n

. By the binomial theorem,

tn= 1 + 1 + 1

2! 1 − 1 n + 1

3! 1 − 1 n

 1 − 2

n + · · · + 1

n! 1 − 1 n

 1 − 2

n · · · 1 − n − 1 n .

Hence tn≤ sn ≤ e for all n ∈ N, so that

lim sup

n→∞

tn≤ e.

Next, if n ≥ m, then

tn≥ 1 + 1 + 1

2! 1 − 1

n + · · · + 1

m! 1 − 1 n

 1 − 2

n · · · 1 − m − 1 n .

By keeping m fixed and letting n → ∞ on both sides, we get lim inf

n→∞ tn ≥ 1 + 1 + 1

2! + · · · + 1

m! =⇒ lim inf

n→∞ tn ≥ sm ∀ m ∈ N.

Letting m → ∞, we finally get e ≤ lim inf

n→∞ tn ≤ lim sup

n→∞

tn≤ e =⇒ e = lim

n→∞tn = lim

n→∞(1 + 1 n)n.

Remark

e − sn = 1

(n + 1)! + 1

(n + 2)! + . . .

< 1

(n + 1)!(1 + 1

n + 1 + 1

(n + 1)2 + . . . )

= 1

(n + 1)!

 1

1 − 1/(n + 1)



= 1 n! n

Theorem e 6∈ Q.

Proof Suppose e = m

n for m, n > 0. Then

0 < n! (e − sn)

| {z }

N

< 1 n < 1

(18)

Recall A series converges if its partial sums converges.

Example 1 + 1 2+ 1

4+ 1 8+ 1

16+ . . . converges because partial sums converge.

Cauchy’s Theorem If a1 ≥ a2 ≥ a2· · · ≥ 0, i.e. monotonically decreasing, then

X

n=1

an con-

verges if and only if

X

k=0

2ka2k = a1+ 2a2+ 4a4 + 8a8+ . . . converges.

Proof Compare sn = a1 + · · · + an and tk = a1 + 2a2 + · · · + 2ka2k. Consider the following grouping of the finite sum:

sn = (a1) + (a2+ a3) + (a4 + · · · + a7) + · · · + an

tk = (a1) + (a2+ a2) + (a4 + · · · + a4) + · · · + (a2k + · · · + a2k)

If tk converges, since sn < tk ∀ n < 2k and ∀ k ∈ N,

=⇒ sn≤ lim

k→∞tk ∀ n ∈ N

=⇒ sn converges since sn is monotonically increasing and bounded above.

On the other hand,

if sn converges, since 2sn> tk− a1 ∀ n ≥ 3 and ∀ 2k < n,

=⇒ 2 lim

n→∞sn ≥ tk− a1 ∀ k ∈ N

=⇒ tk converges since tk− a1 is monotonically increasing and bounded above.

This proves that both series diverge or converge simultaneously.

Theorem Consider P 1

np. Claim is that this converges if p > 1 and diverges if p ≤ 1.

Proof If p ≤ 0, terms do not go to zero, so the series diverges. If p > 0, look at:

X

k

2k 1

(2k)p =X

k

2(1−p)k,

which is geometric. Thus it converges if and only if 21−p< 1. This only happens when 1 − p < 0 and hence p > 1.

Remark We were able to turn a harmonic-like series into a geometric-like series.

Theorem (Root Test) Given P an, let α = lim

n→∞

p|an n|. Then (a) if α < 1, then P an converges;

(b) if α > 1, then P an diverges;

(c) if α = 1, then the test is inconclusive.

(19)

Proof If α < 1, we can choose β so that α < β < 1, and an integer N such that p|an n| < β for n ≥ N, by Theorem 3.17(b).

That is, if n ≥ N, then |an| < βn. Since 0 < β < 1, P βn converges. Therefore, P an converges (absolutely) by the comparison test.

If α > 1, then, again by Theorem 3.17(b), there exists a sequence {nk} such that

nk

q

|ank| → α > 1.

Hence |an| > 1 for infinitely many values of n, and

n→∞lim an6= 0 =⇒ X

an diverges.

Since P 1

n diverges, P 1

n2 converges and α = 1 in both cases, the test is inconclusive if α = 1.

Theorem (Ratio Test) The series P an

(a) converges if lim sup

an+1

an < 1, (b) diverges if

an+1

an

≥ 1 for n ≥ n0, where n0 is some fixed integer.

Proof If lim sup

an+1 an

< 1, there exist β < 1 and an integer N such that

an+1

an

< β < 1 for n ≥ N.

This implies that

|aN +k| < β|aN +k−1| < β2|aN +k−2| < · · · < βk|an| for each k ∈ N.

Therefore,P an converges (absolutely) by the comparison test.

If |an+1| ≥ |an| for n ≥ n0, then

n→∞lim an6= 0 =⇒ X

an diverges.

Definition A power series is of the form

X

k=0

cnzn where cn ∈ C.

Theorem Let α = lim supp|cn n|. Let r = α1. Then the power series converges if |z| < R and diverges |z| > R. We call R the radius of convergence.

Proof Use the root test so consider lim sup

n→∞

p|cn nzn| = lim sup

n→∞

|z|p|cn n| = |z| lim sup

n→∞

p|cn n|.

Notice that this less than 1, and thus converges, when |z| is less than 1 over the limsup.

Definition A series converges absolutely if P |an| converges.

(20)

Theorem P an converges absolutely implies P an converges.

Proof |

m

X

k=n

ak| ≤

m

X

k=n

|ak| < , by the Cauchy Criterion since P |ak| converges.

Example If a series converges, it does not necessarily converge absolutely. Consider the series 1 − 12+1314 + . . . , which converges by alternating series test but does not converge absolutely.

Question If the terms in a convergent series are rearranged, must it converge to same sum? Not all the time, but it does if the series converges absolutely.

Riemann’s Theorem If a series P an converges but not absolutely, then we can form a rear- rangement that has any limsup and liminf you’d like.

Example Rearrange 1 − 12 + 1314 + . . . to converge to 4. We want to the partial sums to converge to 4. Consider the positive terms in sequence that sum to at least 4 (notice the positive terms diverge). Then use a s many negative terms you need to go back, as many positive terms you need to go forward, et cetera. These partial sums converge to 4.

參考文獻

相關文件

• Consider an algorithm that runs C for time kT (n) and rejects the input if C does not stop within the time bound.. • By Markov’s inequality, this new algorithm runs in time kT (n)

– The The readLine readLine method is the same method used to read method is the same method used to read  from the keyboard, but in this case it would read from a 

Proof: For every positive integer n, there are finitely many neighbor- hood of radius 1/n whose union covers K (since K is compact). Collect all of them, say {V α }, and it forms

Note: BCGD has stronger global convergence property (and cheaper iteration) than BCM..1. Every cluster point of the x-sequence is a minimizer

For periodic sequence (with period n) that has exactly one of each 1 ∼ n in any group, we can find the least upper bound of the number of converged-routes... Elementary number

It has been well-known that, if △ABC is a plane triangle, then there exists a unique point P (known as the Fermat point of the triangle △ABC) in the same plane such that it

If x or F is a vector, then the condition number is defined in a similar way using norms and it measures the maximum relative change, which is attained for some, but not all

A subgroup N which is open in the norm topology by Theorem 3.1.3 is a group of norms N L/K L ∗ of a finite abelian extension L/K.. Then N is open in the norm topology if and only if